Đến nội dung

ongngua97 nội dung

Có 289 mục bởi ongngua97 (Tìm giới hạn từ 30-03-2020)



Sắp theo                Sắp xếp  

#527139 $\left\{\begin{matrix} u_{0}=1;...

Đã gửi bởi ongngua97 on 04-10-2014 - 18:01 trong Dãy số - Giới hạn

Cho dãy số   $u_{n}$ được xác định bởi :

$\left\{\begin{matrix} u_{0}=1; u_{1}=-1 & \\ u_{n}=6u_{n-1}+5u_{n-2} & \end{matrix}\right.$

 

Chứng minh rằng $u_{2012}-2010\vdots 2011$




#510223 $2\sqrt{3x^4+x^3}+13(x-\frac{1}{2...

Đã gửi bởi ongngua97 on 01-07-2014 - 21:38 trong Phương trình - hệ phương trình - bất phương trình

Giải bất phương trình:

$2\sqrt{3x^4+x^3}+13(x-\frac{1}{2})^2\geq \frac{29}{4}+2(x-1)\sqrt{x+7x^2}$




#509603 $\left\{\begin{matrix} x^2y^2+2y^2+16=11xy...

Đã gửi bởi ongngua97 on 28-06-2014 - 14:12 trong Phương trình - hệ phương trình - bất phương trình

Giải phương trình:

 

$\left\{\begin{matrix}

x^2y^2+2y^2+16=11xy & \\x^2+2y^2+12y=3xy^2 
 & 

\end{matrix}\right.$




#495463 Lỗi sai lời giải đề thi thử.

Đã gửi bởi ongngua97 on 27-04-2014 - 16:22 trong Bất đẳng thức và cực trị

sai ở dòng 2 từ trên xuống ở hình này nè

Hình gửi kèm

  • Capture2.PNG



#495458 Lỗi sai lời giải đề thi thử.

Đã gửi bởi ongngua97 on 27-04-2014 - 16:19 trong Bất đẳng thức và cực trị

hình

Hình gửi kèm

  • 1.PNG



#495455 Lỗi sai lời giải đề thi thử.

Đã gửi bởi ongngua97 on 27-04-2014 - 16:15 trong Bất đẳng thức và cực trị

Trong bài BĐT của đề thi thử Chuyên Lương Văn Chánh Phú Yên lần 2, mình thấy lời giải của bài  này mà trường đưa ra có vấn đề

ở dòng thứ 2 từ trên xuống ở hình 3, mọi người xem nhé.

 

p/s: mình k biết ghi tiêu đề như thế nào, có gì mong ĐHV góp ý.

Hình gửi kèm

  • Capture3.PNG



#494192 Đề thi olympic Duyên Hải Bắc Bộ năm 2013-2014

Đã gửi bởi ongngua97 on 20-04-2014 - 16:59 trong Thi HSG cấp Tỉnh, Thành phố. Olympic 30-4. Đề thi và kiểm tra đội tuyển các cấp.

mo.png

Môn Toán lớp 10.

Câu 1. Giải phương trình:

$$(6x-3)\sqrt{7-3x}+(15-6x)\sqrt{3x-2}=2\sqrt{-9x^{2}+27x-14}+11.$$

Câu 2. Cho tam giác $ABC (BC<AC)$. Gọi $M$ là trung điểm $AB, AP$ vuông góc với $BC$ tại $P, BQ$ vuông góc với $AC$ tại $Q$. Giả sử đường thẳng $PQ$ cắt $AB$ tại $T$. Chứng minh $TH$ vuông góc $CM$. ($H$ là trực tâm tam giác $ABC$).

Câu 3. Cho hàm số $f: \mathbb{R}\rightarrow \mathbb{R}$ ($\mathbb{R}$ là tập số thực) thỏa mãn:

$$f(f(x))=x^{3}+\frac{3}{4}x, \forall x\in \mathbb{R}$$

Chứng minh tồn tại 3 số thực phân biệt $a,b,c$ sao cho $f(a)+f(b)+f(c)=0$.

Câu 4. Tìm $k$ lớn nhất để bất đẳng thức sau đúng với mọi $a,b,c$, ta có:

$$a^{4}+b^{4}+c^{4}+abc(a+b+c)\geq k(ab+bc+ca)^{3}$$

Câu 5. Tìm số nguyên dương $n$ nhỏ nhất để $2013^{n}-1$ chia hết cho $2^{2014}$.

 
Môn toán 11
Câu 1: Giải hệ phương trình:
$$\left \{ \begin{matrix}2x-2y+\sqrt{x+y+3xy+1}=1\\ \sqrt[3]{3y+1}=8x^2-2y-1\\ x>0\end{matrix}\right.$$
 
Câu 2: Cho dãy $\left ( a_n \right )_{n=1}^{\infty }$ xác định như sau:
$$a_1=1,a_{n+1}=\frac{a_n^3-a_n+10}{5-a_n}, \forall n \geq 1$$
1. Chứng minh dãy hội tụ và tính giới hạn
2. Chứng minh : $\frac{a_1+a_2+...+a_n}{n}< \frac{5-\sqrt{5}}{2}$ với mọi $n\geq 1$
 
Câu 3: Gọi $AD,BE,CF$ là  đường phân giác trong của tam giác $ABC$ vuông tại $A$ . Đoạn thẳng $AD$ cắt $EF$ tại $K$. Đường thẳng qua $K$  song song với $BC$ cắt $AB,AC$ lần lượt tại $M,N$. Chứng minh rằng:
$$MN\geq \frac{2-\sqrt{2}}{2}(AB+AC)$$
 
Câu 4: Tìm tất cả hàm số $f:\mathbb{R}\rightarrow \mathbb{R}$ thỏa mãn :
$$f(x^2+y^2)=xf(x)+yf(y), \forall x,y \in \mathbb{R}$$
 
Câu 5: Cho $100$ số tự nhiên không lớn hơn $100$ có tổng bằng $200$. Chứng minh rằng từ các số đó có thể chọn được ít nhất một bộ các số có tổng bằng $100$.

 

 

 

 

 




#493993 Đề thi chọn HSG toán 10 khu vực duyên hải và đồng bằng bắc bộ 2013-2014

Đã gửi bởi ongngua97 on 19-04-2014 - 22:46 trong Thi HSG cấp Tỉnh, Thành phố. Olympic 30-4. Đề thi và kiểm tra đội tuyển các cấp.

Có đề 11 k nhỉ?




#493924 Bình luận về $(3x+1)\sqrt{2x^2 -1}=5x^2 -\frac{...

Đã gửi bởi ongngua97 on 19-04-2014 - 19:17 trong Phương trình - hệ phương trình - bất phương trình

 

 $(3x+1)\sqrt{2x^2 -1}=5x^2 +\frac{3}{2}x-3$ (1)

Mình mạn phép xin nêu cách giải:

Đặt t=$\sqrt{2x^2 -1}$

Phân tích (1) thành $\alpha (2x^2-1)+ (10-2\alpha )x^2+3x-6+\alpha= 2(3x+1)\sqrt{2x^2 -1}$ (2)

   $\Delta (2)= (9-10\alpha +2\alpha ^2)x^2 +(6-3\alpha )x +1-\alpha (\alpha -6)$ (3)

Để tìm ra mối liên hệ giữa t và x thì $\Delta (2)$ phải phân tích thành bình phương của 1 biểu thức nên $\Delta (3) =(6-3\alpha )^2 -4.\left [ 1-\alpha (\alpha -6) \right](2\alpha ^2 -10\alpha +9)=0$ rồi tìm ra $\alpha =4$ 

Mình thấy phương pháp của mình hơi dài, mong các bạn góp ý thêm. Có cách nào " lụi" $\alpha$ không nhỉ, bởi vào phòng thi chỉ có 18 phút một câu và tâm lí nữa. Mình xin cảm ơn

 

Điều kiện cần là (9 -10a+2a2) và 1-a(a-6) là số chính phương, thử các th a=1,2,3,4 thấy là đúng => chọn 4. :)




#493556 CM tứ giác QEVH nội tiếp

Đã gửi bởi ongngua97 on 17-04-2014 - 20:05 trong Hình học

Cho tg ABC nội tiếp đường tròn tâm O, 3 đường cao AD,BE,CF cắt nhau tại H, BE cắt FD tại S, trung trực SE căt CF tại Q, cắt AD tại V. CM tứ giác QEVH nội tiếp.




#492404 $\frac{1}{x^{2}+y^{2}+2}+...

Đã gửi bởi ongngua97 on 12-04-2014 - 12:46 trong Bất đẳng thức - Cực trị

Cho  x,y,z >0 thoả xy+yz+xz=3

 

Chứng minh rằng  P=$\frac{1}{x^{2}+y^{2}+2}+\frac{1}{y^2+z^2+2}+\frac{1}{x^2+z^2+2}\leq \frac{3}{4}$




#491076 Đề thi chính thức Olympic 30-4 toán 11 lần thứ XX năm 2014

Đã gửi bởi ongngua97 on 06-04-2014 - 17:27 trong Thi HSG cấp Tỉnh, Thành phố. Olympic 30-4. Đề thi và kiểm tra đội tuyển các cấp.

Câu hệ giải ntn z mọi người???




#486372 $f(3x-y+a)=3f(x)-f(y), \forall x,y \in \mathbb{R...

Đã gửi bởi ongngua97 on 09-03-2014 - 11:05 trong Phương trình hàm

Tìm các hàm số $f$ liên tục trên $R$ thoả mãn :

$f(3x-y+a)=3f(x)-f(y), \forall x,y \in \mathbb{R}$

Trong đó a là số thực cho trước.




#478643 Giải bài trên báo Toán học và tuổi trẻ

Đã gửi bởi ongngua97 on 23-01-2014 - 20:45 trong Toán học & Tuổi trẻ

Giải $1$ bài cũng được.

Bạn nên viết tay rồi gửi đến 187B Giảng Võ, Hà Nội ( Qua đường bưu điện)

Nếu không thì bạn viết cách giải ở Word rồi gửi qua email của Báo ấy. 

Bạn có chắc là gởi mail được k vậy?




#478555 cMR:$\frac{1}{a+b+c}\geq \frac{1...

Đã gửi bởi ongngua97 on 22-01-2014 - 22:37 trong Bất đẳng thức và cực trị

Cho a,b,c>1.cMR:$\frac{1}{a+b+c}\geq \frac{1}{3}+\frac{(1-a)(1-b)(1-c)}{3}$

Bài toán sai khi cho a=1.1, b=c=2




#478233 Báo Toán học tuổi trẻ số 439

Đã gửi bởi ongngua97 on 20-01-2014 - 18:02 trong Toán học & Tuổi trẻ

Bạn nào có báo Toán học tuổi trẻ số 439 tháng 1 không, up lên giùm mình với.Chụp ảnh cũng được. Cảm ơn nhiều.:)




#478232 $\frac{a^2}{b}+\frac{b^2}{c...

Đã gửi bởi ongngua97 on 20-01-2014 - 17:58 trong Bất đẳng thức - Cực trị

Cho a,b,c>0 thoả abc=1

 

Chứng minh rằng : $\frac{a^2}{b}+\frac{b^2}{c}+\frac{c^2}{a}\geq a^2+b^2+c^2$

 




#478169 Tứ giác $MPNQ$ là hình gì ?

Đã gửi bởi ongngua97 on 20-01-2014 - 00:35 trong Hình học

Đây là hình em mới vẽ xong.

Hình gửi kèm

  • Capture.PNG



#478168 Tứ giác $MPNQ$ là hình gì ?

Đã gửi bởi ongngua97 on 20-01-2014 - 00:31 trong Hình học


Cho tam giác $ABC$ vuông ở $A$, nửa đường tròn đường kính $AB$ cắt $BC$ tại $D$. Trên cung $AD$ lấy một điểm $E$. Nối $BE$ và kéo dài cắt $AC$ tại $F$.
a) Chứng minh tứ giác $CDEF$ là một tứ giác nội tiếp.
b) Kéo dài $DE$ cắt $AC$ ở $K$. Tia phân giác của góc $CKD$ cắt $EF$ và $CD$ tại $M$ và $N$. Tia phân giác của góc $CBF$ cắt $DE$ và $CF$ tại $P$ và $Q$. Tứ giác $MPNQ$ là hình gì ? Tại sao?
 

a, Ta có $\angle BED=\angle BAD$ (cùng chắn cung nhỏ BD)

          $\angle BAD=\angle ACB$ ( cùng phụ với $\angle ABC$ )

$\Rightarrow \angle BED=ACB$ nên tứ giác CDEF nội tiếp.

 

b. Xét 2 tam giác : $\Delta BPE, \Delta BQC$ có:

$\angle PBE=\angle PBC$ ( do BP là phân giác $\angle DBE$)

$\angle BEP=\angle BCQ$ (chứng minh ở câu a)

$\Rightarrow \Delta BPE\sim BQC$

$\Rightarrow \frac{BE}{PE}=\frac{BC}{QC}$

Mà BP là phân giác $\angle DBE, \angle CBF$ 

 

Nên $\frac{BD}{DP}=\frac{BE}{PE}=\frac{BC}{CQ}=\frac{BF}{FQ}$

 

Do đó $\Delta BDP\sim BFQ$ ( Do có $\angle BDE=\angle BFQ$ vì DEFC nội tiếp)

$\Rightarrow \angle KPQ=\angle BPD=\angle KQB$ nên $\Delta KPQ$ cân tại K

$\Rightarrow$ KN vuông góc và đi qua trung điểm PQ ( Do KN là phân giác $\angle PKQ$ 

Tương tự PQ vuông góc và đi qua trung điểm MN

Vậy MNPQ là hình thoi.

Hình gửi kèm

  • Capture.PNG



#478058 $a,b \epsilon [o,1]$

Đã gửi bởi ongngua97 on 19-01-2014 - 17:05 trong Đại số

Mọi người cho em hỏi cách viết này có ý nghĩa gì vậy ạ :

 $a,b \epsilon [0,1]$

 ------------------------------------------

P/s: Học toán bấy lâu nay mà hôm nay tự nhiên gặp cái này không biết...thật là ngại quá !

 

Nó đồng nghĩa với $0\leq a,b\leq 1$




#477791 $\frac{a^2}{b}+\frac{b^2}{c...

Đã gửi bởi ongngua97 on 17-01-2014 - 22:28 trong Bất đẳng thức và cực trị

Mình làm thế này không biết có đúng không, nhờ các bạn xem giùm.

Ta có:

$\frac{a^2}{b}+\frac{b^2}{c}+\frac{c^2}{a}=a^{3}c+b^{3}a+c^{3}b$   vì abc=1

Giả sử $a\geq b\geq c\Rightarrow \left\{\begin{matrix} a^{2}\geq b^{2}\geq c^{2} & \\ ab\geq bc\geq ca & \end{matrix}\right.$

Áp dụng BĐT Trê bư sép, ta có:

$3(a^{3}c+b^{3}a+c^{3}b)\geq (ab+bc+ca)(a^{2}+b^{2}+c^{2})\geq 3\sqrt[3]{a^{2}b^{2}c^{2}}.(a^{2}+b^{2}+c^{2})=3(a^{2}+b^{2}+c^{2})$

Không đúng vì không thể giả sử $a\geq b\geq c$ vì BĐT trên hoán vị chứ không đối xứng.




#477779 $\frac{a^2}{b}+\frac{b^2}{c...

Đã gửi bởi ongngua97 on 17-01-2014 - 21:59 trong Bất đẳng thức và cực trị

áp dụng bđt AM-GM ta có  : $\frac{a^{2}}{b}+a^{2}b\geq 2a^{2}$

chứng minh  ta có  : $\sum \frac{a^{2}}{b}+\sum a^{2}b\geq \sum 2a^{2}$

 ta chỉ cần chứng minh :  $\sum a^{2}b\leq \sum a^{2}$ (1)

ta có

$a+b+c\geq 3\sqrt[3]{abc}=3$ (2)

giả sử (1) đúng , từ (1)(2) suy ra

$(a+b+c)(a^{2}+b^{2}+c^{2})\geq 3(a^{2}b+b^{2}c+c^{2}a)$

$\Rightarrow \sum a^{3}+\sum ab^{2}\geq 2\sum a^{2}b$ (3)

mà ta có

áp dụng bđt AM-GM 

$a^{3}+ ab^{2}\geq 2 a^{2}b$

 chứng minh tương tự suy ra (3) đúng

do (3) đúng nên suy ra (1) đúng

vậy được đpcm

Cách chứng minh của em là không đúng, pp của em có lẽ là biến đổi tương đương để được bđt đúng nhưng khi nhân (2) với (3) là mệnh đề kéo theo chứ k phải tương đương.




#477683 $\frac{a^2}{b}+\frac{b^2}{c...

Đã gửi bởi ongngua97 on 17-01-2014 - 16:36 trong Bất đẳng thức và cực trị

Cho a,b,c>0 thoả abc=1

 

Chứng minh rằng : $\frac{a^2}{b}+\frac{b^2}{c}+\frac{c^2}{a}\geq a^2+b^2+c^2$




#477614 $P=\frac{x^2y}{z^3}+\frac{y^2z}...

Đã gửi bởi ongngua97 on 16-01-2014 - 21:26 trong Bất đẳng thức và cực trị

Cho x,y,z>0

Tìm giá trị nhỏ nhất biểu thức :

 

$P=\frac{x^2y}{z^3}+\frac{y^2z}{x^3}+\frac{z^2x}{y^3}+\frac{4xyz}{xy^2+yz^2+zx^2}$

 

P/s: không biết có ai có cách hay hơn cho bài này không. :icon10:  >:)  :icon12:




#477052 $(a^{2}+b^{2}+c^{2})^{2}+3(a^...

Đã gửi bởi ongngua97 on 13-01-2014 - 12:40 trong Bất đẳng thức - Cực trị

Cho a, b, c dương thỏa mãn : a+b+c=3. CMR: $(a^{2}+b^{2}+c^{2})^{2}+3(a^{2}b^{2}+b^{2}c^{2}+c^{2}a^{2})\geqslant 6(a^{2}+b^{2}+c^{2})$

 

ta cần chứng minh: $\left ( a^2+b^2+c^2 \right )\geq 3(a^2+b^2+c^2)$

và: $\left ( a^2b^2+b^2c^2+c^2a^2 \right )\geq 3(a^2+b^2+c^2)$

giã sử : $a\geq b\geq c>0$ ta có:

$(a^2+b^2+c^2)^2=\sum a^2.\sum a^2\geq 3\sum a^2$  (*)  $theo Bunhia$

và ta cũng có: $\sum \left ( a^2b^2 +1\right )\geq 2\sum ab$

nặt khác ta cũng luôn có: $ab\geq a\Rightarrow \sum ab\geq \sum a$

từ đây suy ra (*) đúng dẫn đến ĐPCM

 

sai rồi

 

Bài toán sai với a=0.9, b=0.99, c=1.11

Lần sau bạn nhớ chú ý đề bài.